Vous êtes sur la page 1sur 19

REAL PROPERTY TAXATION

SEC. OF FINANCE v. ILARDE


G.R. No. 121782. May 9, 2005
CHICO-NAZARIO, J.:

Facts: Cabaluna with his wife owns several real property located in Iloilo City. Cabaluana is the Regional Director of Regional Office No.
VI of the Department of Finance in Iloilo City. After his retirement, there are tax delinquencies on his properties; he paid the amount under
protest contending that the penalties imposed to him are in excess than that provided by law. After exhausting all administrative remedies,
he filed a suit before the RTC which found that Section 4(c) of Joint Assessment Regulation No. 1-85 and Local Treasury Regulation No.
2-85 issued on August 1, 1985 by respondent Secretary (formerly Minister) of Finance is null and void; (2) declaring that the penalty that
should be imposed for delinquency in the payment of real property taxes should be two per centum on the amount of the delinquent tax
for each month of delinquency or fraction thereof, until the delinquent tax is fully paid but in no case shall the total penalty exceed twenty-
four per centum of the delinquent tax as provided for in Section 66 of P.D. 464 otherwise known as the Real Property Tax Code.

Issue: Whether or not the then Ministry of Finance could legally promulgate Regulations prescribing a rate of penalty on delinquent taxes
other than that provided for under Presidential Decree (P.D.) No. 464, also known as the Real Property Tax Code.

Held: The Ministry of Finance now Secretary of Finance cannot promulgate regulations prescribing a rate of penalty on delinquent taxes.
The Court ruled that despite the promulgation of E.O. No. 73, P.D. No. 464 in general and Section 66 in particular, remained to be good
law. To accept the Secretary’s premise that E.O. No. 73 had accorded the Ministry of Finance the authority to alter, increase, or modify
the tax structure would be tantamount to saying that E.O. No. 73 has repealed or amended P.D. No. 464. Repeal of laws should be made
clear and expressed. Repeals by implication are not favored as laws are presumed to be passed with deliberation and full knowledge of
all laws existing on the subject. Such repeals are not favored for a law cannot be deemed repealed unless it is clearly manifest that the
legislature so intended it. Assuming argumenti that E.O. No. 73 has authorized the petitioner to issue the objected Regulations, such
conferment of powers is void for being repugnant to the well-encrusted doctrine in political law that the power of taxation is generally
vested with the legislature. Thus, for purposes of computation of the real property taxes due from private respondent for the years 1986
to 1991, including the penalties and interests, is still Section 66 of the Real Property Tax Code of 1974 or P.D. No. 464. The penalty that
ought to be imposed for delinquency in the payment of real property taxes should, therefore, be that provided for in Section 66 of P.D.
No. 464, i.e., two per centum on the amount of the delinquent tax for each month of delinquency or fraction thereof but “in no case shall
the total penalty exceed twenty-four per centum of the delinquent tax.”

Dispositive: WHEREFORE, the instant petition is hereby DENIED and the order dated 28 August 1995 in Civil Case No. 21207 rendered
by respondent Judge Ricardo M. Ilarde of the Regional Trial Court, 6th Judicial Region, Branch 26, Iloilo City, is hereby AFFIRMED with
MODIFICATION that the real property tax payable by private respondent Cipriano P. Cabaluna, Jr., for the year 1992 shall be based on
the Local Government Code of 1991. No costs.
----------------------------------------------xxx---------------------------------------------
BENGUET CORPORATION VS. CENTRAL BOARD OF ASSESSMENT APPEALS ET AL
G.R. No. 106041, January 29, 1993

FACTS:
In 1985, the Provincial Assessor of Zambales assessed petitioner’s tailings dam and land thereunder as taxable improvements over its
protest. The realty tax assessment amounted to P11,319,304.00. The assessment was appealed to the Board of Assessment Appeals of
the Province of Zambales. The appeal was dismissed on the ground of the petitioner's failure to pay the realty taxes that fell due during
the pendency of the appeal.

On appeal, respondent CBAA reversed the dismissal of the appeal but agreed that the tailings dam and the lands submerged were
subject to realty tax.

Petitioner’s contention
While petitioner does not dispute that the tailings dam may be considered realty within the meaning of Article 415 of the Civil Code, it
insists, however, that the dam cannot be subjected to realty tax as a separate and independent property because it does not constitute
an "assessable improvement" on the mine although a considerable sum may have been spent in constructing and maintaining it.

OSG’s contention
OSG argued that the dam is an assessable improvement because it enhances the value and utility of the mine. The primary function of
the dam is to receive, retain and hold the water coming from the operations of the mine, and it also enables the petitioner to impound
water, which is then recycled for use in the plant.

ISSUES:
1. Whether or not the tailings dam is subject to realty tax?
2. Whether or not it can be considered as immovable property?
HELD:
Yes. Petition is denied.

It would appear that whether a structure constitutes an improvement so as to partake of the status of realty would depend upon the degree
of permanence intended in its construction and use. The expression "permanent" as applied to an improvement does not imply that the
improvement must be used perpetually but only until the purpose to which the principal realty is devoted has been accomplishe d. It is
sufficient that the improvement is intended to remain as long as the land to which it is annexed is still used for the said purpose.

The Real Property Tax Code does not carry a definition of "real property" and simply says that the realty tax is imposed on "real property,
such as lands, buildings, machinery and other improvements affixed or attached to real property." In the absence of such a definition, we
apply Article 415 of the Civil Code, the pertinent portions of which state:
Art. 415. The following are immovable property.
(1) Lands, buildings and constructions of all kinds adhered to the soil;
xxx xxx xxx
(3) Everything attached to an immovable in a fixed manner, in such a way that it cannot be separated therefrom without breaking the
material or deterioration of the object.

Section 2 of C.A. No. 470 or the Assessment Law provides that the realty tax is due "on the real property, including land, buildings,
machinery and other improvements" not specifically exempted in Section 3 thereof. Thus, the tailings dam does not fall under any of the
classes of exempt real properties enumerated.
Is the tailings dam an improvement on the mine? Section 3(k) of the Real Property Tax Code defines improvement as follows:
(k) Improvements — is a valuable addition made to property or an amelioration in its condition, amounting to more than mere repairs or
replacement of waste, costing labor or capital and intended to enhance its value, beauty or utility or to adopt it for new or further purposes.

The Court held that the subject dam falls within the definition of an "improvement" because it is permanent in character and it enhances
both the value and utility of petitioner's mine. Moreover, the immovable nature of the dam defines its character as real property under
Article 415 of the Civil Code and thus makes it taxable under Section 38 of the Real Property Tax Code.

Likewise, the Court gave respect to the conclusions of quasi-judicial agencies like the CBAA, which, because of the nature of its functions
and its frequent exercise, has developed expertise in the resolution of assessment problems. The only exception to this rule is where it
is clearly shown that the administrative body has committed grave abuse of discretion.
----------------------------------------xxx------------------------------------------
NATIONAL DEVELOPMENT CORP v. CEBU CITY
----------------------------------------xxx------------------------------------------
PROVINCE OF TARLAC v. JUDGE ALCANTARA
----------------------------------------xxx-----------------------------------------
SHARES OF LGUs IN NATIONAL TAXES

PIMENTEL v. AGUIRRE
G.R. No. 132988. July 19, 2000
PANGANIBAN, J.:

Facts:

In 1997, President Ramos issued AO 372 which: (1) required all government departments and agencies, including SUCs, GOCCs and
LGUs to identify and implement measures in FY 1998 that will reduce total expenditures for the year by at least 25% of authorized regular
appropriations for non--personal services items (Section 1) and (2) ordered the withholding of 10% of the IRA to LGUs (Section 4) . On
10 December 1998, President Estrada issued AO 43, reducing to 5% the amount of IRA to be withheld from LGU.

Issues:

1. Whether or not the president committed grave abuse of discretion in ordering all LGUS to adopt a 25% cost reduction program in
violation of the LGU'S fiscal autonomy

2. Whether Section 4 of the same issuance, which withholds 10 percent of their internal revenue allotments, are valid exercises of the
President's power of general supervision over local governments

Held:

1. Section 1 of AO 372 does not violate local fiscal autonomy. Local fiscal autonomy does not rule out any manner of national government
intervention by way of supervision, in order to ensure that local programs, fiscal and otherwise, are consistent with national goals.
Significantly, the President, by constitutional fiat, is the head of the economic and planning agency of the government, primarily
responsible for formulating and implementing continuing, coordinated and integrated social and economic policies, plans and programs
for the entire country. However, under the Constitution, the formulation and the implementation of such policies and programs are subject
to "consultations with the appropriate public agencies, various private sectors, and local government units." The President cannot do so
unilaterally.

Consequently, the Local Government Code provides:


"x x x [I]n the event the national government incurs an unmanaged public sector deficit, the President of the Philippines is hereby
authorized, upon the recommendation of [the] Secretary of Finance, Secretary of the Interior and Local Government and Secretary of
Budget and Management, and subject to consultation with the presiding officers of both Houses of Congress and the presidents of the
liga, to make the necessary adjustments in the internal revenue allotment of local government units but in no case shall the allotment be
less than thirty percent (30%) of the collection of national internal revenue taxes of the third fiscal year preceding the current fiscal year
x x x."
There are therefore several requisites before the President may interfere in local fiscal matters: (1) an unmanaged public sector deficit of
the national government; (2) consultations with the presiding officers of the Senate and the House of Representatives and the presidents
of the various local leagues; and (3) the corresponding recommendation of the secretaries of the Department of Finance, Interior and
Local Government, and Budget and Management. Furthermore, any adjustment in the allotment shall in no case be less than thirty
percent (30%) of the collection of national internal revenue taxes of the third fiscal year preceding the current one.

Petitioner points out that respondents failed to comply with these requisites before the issuance and the implementation of AO 372. At
the very least, they did not even try to show that the national government was suffering from an unmanageable public sector deficit.
Neither did they claim having conducted consultations with the different leagues of local governments. Without these requisites, the
President has no authority to adjust, much less to reduce, unilaterally the LGU's internal revenue allotment.

AO 372, however, is merely directory and has been issued by the President consistent with his power of supervision over local
governments. It is intended only to advise all government agencies and instrumentalities to undertake cost-reduction measures that will
help maintain economic stability in the country, which is facing economic difficulties. Besides, it does not contain any sanction in case of
noncompliance. Being merely an advisory, therefore, Section 1 of AO 372 is well within the powers of the President. Since it is not a
mandatory imposition, the directive cannot be characterized as an exercise of the power of control.

2. Section 4 of AO 372 cannot be upheld. A basic feature of local fiscal autonomy is the automatic release of the shares of LGUs in the
national internal revenue. This is mandated by no less than the Constitution. The Local Government Code specifies further that the
release shall be made directly to the LGU concerned within five (5) days after every quarter of the year and "shall not be subject to any
lien or holdback that may be imposed by the national government for whatever purpose." As a rule, the term "shall" is a word of command
that must be given a compulsory meaning. The provision is, therefore, imperative.
--------------------------------------------------XXX-------------------------------------------------
ABATEMENT OF NUISANCE

ESTATE OF GREGORIA FRANCISCO v. COURT OF APPEALS


G.R. No. 95279, 25 July 1991, 199 SCRA 595
Melencio-Herrera, J.

FACTS: Basilan Municipal Mayor Benjamin Valencia summarily ordered the demolition of an antiquated and dilapidated quonset
warehouse situated in Port Area, Strong Boulevard, Isabela, Basilan, outside the zone for warehouses. The legal possessor of the quonset
sought the prohibition of the Order but was denied by the RTC. The CA originally overturned the RTC but subsequently reversed itself.
In question in this case is the validity of such order by the Municipal Mayor, which was in effect an abatement of nuisance, without prior
judicial authority.

ISSUE: Whether or not Respondent Mayor could summarily and extra-judicially order the demolition of petitioner's quonset building.

HELD: NO

Ordinance No. 147 relied upon by Respondents should not be interpreted as authorizing the summary removal of a non-conforming
building by the municipal government. For if it does, it must be struck down for being in contravention of the requirements of due
process, as originally held by the Court of Appeals.

Moreover, the enforcement and administration of the provisions of the Ordinance resides with the Zoning Administrator. It is said official
who may call upon the City Fiscal to institute the necessary legal proceedings to enforce the provisions of the Ordinance. And any person
aggrieved by the decision of the Zoning Administrator regarding the enforcement of the Ordinance may appeal to the Board of Zoning
Appeals.

Violation of a municipal ordinance neither empowers the Municipal Mayor to avail of extra-judicial remedies. On the contrary, the Local
Government Code imposes upon him the duty "to cause to be instituted judicial proceedings in connection with the violation of ordinances"
(Local Government Code, Sec. 141 [2] [t]).

Respondents cannot seek cover under the general welfare clause authorizing the abatement of nuisances without judicial
proceedings, which applies only to a nuisance per se or one which affects the immediate safety of persons and property and may be
summarily abated under the undefined law of necessity (Monteverde v. Generoso, 52 Phil. 123 [1982]). The storage of copra in the
quonset building is a legitimate business. By its nature, it cannot be said to be injurious to rights of property, of health or of comfort of the
community. If it be a nuisance per accidens it may be so proven in a hearing conducted for that purpose. It is not per se a nuisance
warranting its summary abatement without judicial intervention.

The provincial governor, district engineer or district health officer is not authorized to destroy private property consisting of dams and
fishponds summarily and without any judicial proceedings whatever under the pretense that such private property constitutes a nuisance.
A dam or a fishery constructed in navigable rivers is not a nuisance per se. A dam or fishpond may be a nuisance per accidens where it
endangers or impairs the health or depreciates property by causing water to become stagnant. (Monteverde v. Generoso, supra).

While the Sangguniang Bayan may provide for the abatement of a nuisance (Local Government Code, Sec. 149 [ee]), it can not declare
a particular thing as a nuisance per se and order its condemnation. The nuisance can only be so adjudged by judicial determination.

[Municipal councils] do not have the power to find as a fact that a particular thing is a nuisance when such thing is not a nuisance per se
nor can they authorize the extra judicial condemnation and destruction of that as a nuisance which, in its nature, situation or use is not
such. These things must be determined in the ordinary courts of law. In the present case, . . . the ice factory of the plaintiff is not a
nuisance per se. It is a legitimate industry . . . . If it be in fact a nuisance due to the manner of its operation, that question cannot be
determined by a mere resolution of the board. The petitioner is entitled to a fair and impartial heating before a judicial tribunal. (Iloilo Cold
Storage v. Municipal Council, 24 Phil. 47 [1913]).

Petitioner was in lawful possession of the lot and quonset building by virtue of a permit from the Philippine Ports Authority (Port of
Zamboanga) when demolition was effected. It was not squatting on public land. Its property was not of trifling value. It was entitled to an
impartial hearing before a tribunal authorized to decide whether the quonset building did constitute a nuisance in law. There was no
compelling necessity for precipitate action. It follows then that respondent public officials of the Municipality of Isabela, Basilan,
transcended their authority in abating summarily petitioner's quonset building. They had deprived petitioner of its property without due
process of law. The fact that petitioner filed a suit for prohibition and was subsequently heard thereon will not cure the defect, as opined
by the Court of Appeals, the demolition having been a fait accompli prior to hearing and the authority to demolish without a judicial order
being a prejudicial issue.

NOTES:

“Nuisances are of two classes: Nuisances per se and per accidens. As to the first, since they affect the immediate safety of persons
and property, they may be summarily abated under the undefined law of necessity. But if the nuisance be of the second class, even the
municipal authorities, under their power to declare and abate nuisances, would not have the right to compel the abatement of a particular
thing or act as a nuisance without reasonable notice to the person alleged to be maintaining or doing the same of the time and place of
hearing before a tribunal authorized to decide whether such a thing or act does in law constitute a nuisance.” (Monteverde v. Generoso,
52 Phil. 123 (1982), citing Iloilo Ice and Cold Storage Co. vs. Municipal Council of Iloilo [{1913}, 24 Phil., 471])

“Petitioner's business could not be considered a nuisance which respondent municipality could summarily abate in the guise of exercising
its police powers. The abatement of a nuisance without judicial proceedings is possible only if it is a nuisance per se. A gas station is not
a nuisance per se or one affecting the immediate safety of persons and property,17 hence, it cannot be closed down or transferred
summarily to another location.” (PARAYNO v. JOVELLANOS, G.R. No. 148408, 14 July 2006 citing Monteverde v. Generoso, 52 Phil.
123 (1982)
------------------------------------------XXX-------------------------------------------
TECHNOLOGY DEVELOPERS, INC v. CA
G.R. No. 94759, Jan. 21, 1991, 201 SCRA

FACTS:

Technology Developers, a corporation engaged in the manufacture and export of charcoal briquette, received a letter from acting mayor
Pablo Cruz: 1) ordering the full cessation of its plant in Guyong, Sta. Maria, Bulacan until further order, and 2) requesting its Plant Manager
to bring before the office of the mayor its building permit, mayor's permit, and Region III--Pollution of Environment and Natural Resources
Anti--Pollution Permit.

Technology Developers undertook to comply with the request to produce the required documents. It sought to secure the Region III-
Pollution of Environment and Natural Resources Anti--Pollution Permit although prior to the operation of the plant, a Temporary Permit to
Operate Air Pollution Installation was issued to it. Petitioners also sent its representatives to the office of the mayor to secure a mayor’s
permit but were not entertained.

Eventually, the acting mayor ordered that the plant premises be padlocked, effectively causing the stoppage of operation. This was done
without previous and reasonable notice.

Technology Developers then instituted an action for certiorari, prohibition and mandamus with preliminary injunction against the acting
mayor with Bulacan RTC, alleging that the closure order was issued in grave abuse of discretion.
The RTC found that the issuance of the writ of preliminary mandatory injunction was proper, ordering the acting mayor to immediately
revoke his closure order and allow Technology Developers to resume its normal business operations until the case has been adjudicated
on the merits.

Upon MR, the Provincial Prosecutor presented evidence as to the allegation that "Due to the manufacturing process and nature of raw
materials used, the fumes coming from the factory may contain particulate matters which are hazardous to the health of the people. As
such, the company should cease operating until such a time that the proper air pollution device is installed and operational."

Reassessing the evidence, the RTC set aside its order granted the writ of preliminary mandatory injunction. The CA denied Technology
Developer's petition for certiorari for lack of merit.

ISSUE:

W/N the acting mayor had a legal ground for ordering the stoppage of Technology Developer

HELD:

YES. The following circumstances militate against the maintenance of the writ of preliminary injunction sought by petitioner:

1. No mayor's permit had been secured. While it is true that the matter of determining whether there is a pollution of the environment that
requires control if not prohibition of the operation of a business is essentially addressed to the Environmental Management Bureau of the
Department of Environment and Natural Resources, it must be recognized that the mayor of a town has as much responsibility to protect
its inhabitants from pollution, and by virtue of his police power, he may deny the application for a permit to operate a business or otherwise
close the same unless appropriate measures are taken to control and/or avoid injury to the health of the residents of the community from
the emissions in the operation of the business.

2. The Acting Mayor called the attention of petitioner to the pollution emitted by the fumes of its plant whose offensive odor "not only
pollute the air in the locality but also affect the health of the residents in the area," so that petitioner was ordered to stop its operation until
further orders.

3. This action of the Acting Mayor was in response to the complaint of the residents of Barangay Guyong, Sta. Maria, Bulacan, directed
to the Provincial Governor through channels.

4. The closure order of the Acting Mayor was issued only after an investigation was made by Marivic Guina who in her report observed
that the fumes emitted by the plant goes directly to the surrounding houses and that no proper air pollution device has been installed.

5. Petitioner failed to produce a building permit from the municipality of Sta. Maria, but instead presented a building permit issued by an
official of Makati on March 6, 1987.

6. While petitioner was able to present a temporary permit to operate by the then National Pollution Control Commission on December
15, 1987, the permit was good only up to May 25, 1988. Petitioner had not exerted any effort to extend or validate its permit much less to
install any device to control the pollution and prevent any hazard to the health of the residents of the community.

Court takes note of the plea of petitioner focusing on its huge investment in this dollar-earning industry. It must be stressed however, that
concomitant with the need to promote investment and contribute to the growth of the economy is the equally essential imperative of
protecting the health, nay the very lives of the people, from the deleterious effect of the pollution of the environment.

● The well-known rule is that the matter of issuance of a writ of preliminary injunction is addressed to the sound judicial discretion of the
trial court and its action shall not be disturbed on appeal unless it is demonstrated that it acted without jurisdiction or in excess of
jurisdiction or otherwise, in grave abuse of its discretion. By the same token the court that issued such a preliminary relief may recall or
dissolve the writ as the circumstances may warrant.
Petition denied.
------------------------------------------------------XXX--------------------------------------------------------
LAGUNA LAKE DEVELOPMENT AUTHORITY v. CA
G.R. Nos. 120865-71 December 7, 1995

Facts:
The Laguna Lake Development Authority (LLDA) was created through RA No. 4850 in order to execute the policy towards environmental
protection and sustainable development so as to accelerate the development and balanced growth of the Laguna Lake area and the
surrounding provinces and towns.
Upon implementation of RA 7160 (Local Government Code of 1991), the municipalities assumed exclusive jurisdiction & authority to issue
fishing privileges within their municipal waters since Sec.149 thereof provides: “Municipal corporations shall have the authority to grant
fishery privileges in the municipal waters and impose rental fees or charges therefore…” Big fishpen operators took advantage of the
occasion to establish fishpens & fish cages to the consternation of the LLDA.
The implementation of separate independent policies in fish cages & fish pen operation and the indiscriminate grant of fishpen permits
by the lakeshore municipalities have saturated the lake with fishpens, thereby aggravating the current environmental problems and
ecological stress of Laguna Lake.
The LLDA then served notice to the general public that:
(1) fishpens, cages & other aqua-culture structures unregistered with the LLDA as of March 31, 1993 are declared illegal;
(2) those declared illegal shall be subject to demolition by the Presidential Task Force for Illegal Fishpen and Illegal Fishing; and
(3) owners of those declared illegal shall be criminally charged with violation of Sec.39-A of RA 4850 as amended by PD 813.
A month later, the LLDA sent notices advising the owners of the illegally constructed fishpens, fishcages and other aqua-culture structures
advising them to dismantle their respective structures otherwise demolition shall be effected.

Issue
Which agency of the Government — the Laguna Lake Development Authority or the towns and municipalities comprising the region —
should exercise jurisdiction over the Laguna Lake and its environs insofar as the issuance of permits for fishery privileges is concerned?

Held
LLDA has jurisdiction over such matters because the charter of the LLDA prevails over the Local Government Code of 1991.
The said charter constitutes a special law, while the latter is a general law.
The Local Government Code of 1991, has not repealed the provisions of the charter of the Laguna Lake Development Authority, Republic
Act No. 4850, as amended.
Thus, the Authority has the exclusive jurisdiction to issue permits for the enjoyment of fishery privileges in Laguna de Bay to the exclusion
of municipalities situated therein and the authority to exercise such powers as are by its charter vested on it.
In addition, the charter of the LLDA embodies a valid exercise of police power for the purpose of protecting and developing the Laguna
Lake region, as opposed to the Local Government Code, which grants powers to municipalities to issue fishing permits for revenue
purposes.
Thus, it has to be concluded that the charter of the LLDA should prevail over the Local Government Code of 1991 on matters affecting
Laguna de Bay.
-------------------------------------------------XXX-------------------------------------------------
POWER OF EMINENT DOMAIN

Moday vs CA
Date: February 20, 1997
Petitioners: Percival Moday, Zotico Moday and Leonora Moday
Respondents: CA, Judge Evangelista Yuipco, and Municipality of Bunawan

Ponente: Romero

Facts: The Sangguniang Bayan of the Municipality of Bunawan in Agusan del Sur passed Resolution No. 43-89, "Authorizing the Municipal
Mayor to Initiate the Petition for Expropriation of a One (1) Hectare Portion of Lot No. 6138-Pls-4 Along the National Highway Owned by
Percival Moday for the Site of Bunawan Farmers Center and Other Government Sports Facilities." The Resolution was approved by
Mayor Anuncio Bustillo and was transmitted to the Sangguniang Panlalawigan for its approval.
The Sangguniang Panlalawigan disapproved said Resolution and returned it with the comment that "expropriation is unnecessary
considering that there are still available lots in Bunawan for the establishment of the government center."
The municipality filed a petition for eminent domain against Percival Moday before the RTC. The municipality then filed a motion
to take or enter upon the possession of the land upon deposit with the municipal treasurer of the required amount. The RTC granted the
motion. It ruled that the Sangguniang Panlalawigan's failure to declare the resolution invalid leaves it effective. It added that the duty of
the Sangguniang Panlalawigan is merely to review the ordinances and resolutions passed by the Sangguniang Bayan under Section 208
(1) of B.P. Blg. 337, old Local Government Code and that the exercise of eminent domain is not one of the acts enumerated in Section
19 requiring the approval of the Sangguniang Panlalawigan.
Petitioners elevated the case in a petition for certiorari before the CA. The CA held that the public purpose for the expropriation
is clear from Resolution No. 43-89 and that since the Sangguniang Panlalawigan of Agusan del Sur did not declare Resolution No. 43-
89 invalid, expropriation of petitioners' property could proceed. Meanwhile, the Municipality had erected three buildings on the subject
property: the Association of Barangay Councils (ABC) Hall, the Municipal Motorpool, both wooden structures, and the Bunawan Municipal
Gymnasium, which is made of concrete.
In the instant petition for review, petitioner seeks the reversal of the decision and resolution of the CA and a declaration that Resolution
No. 43-89 of the Municipality of Bunawan is null and void.

Issue: WON a municipality may expropriate private property by virtue of a municipal resolution which was disapproved by the
Sangguniang Panlalawigan.

Held: Yes
Ratio: Eminent domain, the power which the Municipality of Bunawan exercised in the instant case, is a fundamental State power that is
inseparable from sovereignty. It is government's right to appropriate, in the nature of a compulsory sale to the State, private property for
public use or purpose. Inherently possessed by the national legislature, the power of eminent domain may be validly delegated to local
governments, other public entities and public utilities. For the taking of private property by the government to be valid, the taking must be
for public use and there must be just compensation.
The Municipality's power to exercise the right of eminent domain is not disputed as it is expressly provided for BP 337, the local
Government Code in force at the time expropriation proceedings were initiated. What petitioners question is the lack of authority of the
municipality to exercise this right since the Sangguniang Panlalawigan disapproved Resolution No. 43-89.
The Sangguniang Panlalawigan's disapproval of Resolution No. 43-89 is an infirm action which does not render said resolution null and
void. The law, Section 153 of B.P. Blg. 337, grants the Sangguniang Panlalawigan the power to declare a municipal resolution invalid on
the sole ground that it is beyond the power of the Sangguniang Bayan or the Mayor to issue.
Velazco v. Blas: The only ground upon which a provincial board may declare any municipal resolution, ordinance, or order invalid is
when such resolution, ordinance, or order is "beyond the powers conferred upon the council or president making the same." Absolutely
no other ground is recognized by the law. A strictly legal question is before the provincial board in its consideration of a municipal
resolution, ordinance, or order. The provincial disapproval of any resolution, ordinance, or order must be premised specifically upon the
fact that such resolution, ordinance, or order is outside the scope of the legal powers conferred by law. If a provincial board passes these
limits, it usurps the legislative function of the municipal council or president. Such has been the consistent course of executive authority.
Thus, the Sangguniang Panlalawigan was without the authority to disapprove Municipal Resolution No. 43-89 for the Municipality of
Bunawan clearly has the power to exercise the right of eminent domain and its Sangguniang Bayan the capacity to promulgate sa id
resolution, pursuant to the earlier-quoted Section 9 of B.P. Blg. 337. Perforce, it follows that Resolution No. 43-89 is valid and binding
and could be used as lawful authority to petition for the condemnation of petitioners' property.
As regards the accusation of political oppression, it is alleged that Moday incurred the ire of then Mayor Bustillo when he refused to
support the latter's candidacy for mayor in previous elections. Petitioners claim that then incumbent Mayor Bustillo used the expropriation
to retaliate by expropriating their land even if there were other properties belonging to the municipality and available for the purpose.
Specifically, they allege that the municipality owns a vacant seven-hectare property adjacent to petitioners' land, evidenced by a sketch
plan.
The limitations on the power of eminent domain are that the use must be public, compensation must be made and due process of law
must be observed. The Supreme Court, taking cognizance of such issues as the adequacy of compensation, necessity of the taking and
the public use character or the purpose of the taking, has ruled that the necessity of exercising eminent domain must be genuine and of
a public character. Government may not capriciously choose what private property should be taken.
----------------------------------------XXX--------------------------------------
G.R. NO. 103125, MAY 17, 1993
PROVINCE OF CAMARINES SUR
VS COURT OF APPEALS
PONENTE: QUIASON

Facts:
This is an appeal for certiorari on the decision on the issue on whether the expropriation of agricultural lands by LGU is subject to prior
approval of the DAR.

December 1988, Sangguniang Panlalawigan of CamSur authorized the provincial governor to purchase or expropriate property
contiguous to the provincial capitol site in order to establish a pilot farm for non-food and non-traditional agricultural crops and a housing
project for provincial government employees.

Pursuant to the resolution, Gov. Villafuerte filed two separate cases for expropriation against Ernesto San Joaquin and Efren San Joaquin.
Upon motion for the issuance of writ or possession, San Joaquins failed to appear at the hearing.

San Joaquins later moved to dismiss the complaints on the ground of inadequacy of the price offered for their property. The court denied
the motion to dismiss and authorized the province to take possession of the properties.

San Joaquins filed for motion for relief, but denied as well. In their petition. Asked by the CA, Solicitor General stated that there is no need
for the approval of the president for the province to expropriate properties, however, the approval of the DAR is needed to convert the
property from agricultural to non-agricultural (housing purpose).

CA set aside the decision of the trial court suspending the possession and expropriation of the property until th province has acquired the
approval of DAR. Hence, this petition.

Ruling:
The rules on conversion of agricultural lands found in Section 4 (k) and 5 (1) of Executive Order No. 129-A, Series of 1987, cannot be
the source of the authority of the Department of Agrarian Reform to determine the suitability of a parcel of agricultural land for the purpose
to which it would be devoted by the expropriating authority. While those rules vest on the Department of Agrarian Reform the exclusive
authority to approve or disapprove conversions of agricultural lands for residential, commercial or industrial uses, such authority is limited
to the applications for reclassification submitted by the land owners or tenant beneficiaries.
To sustain the Court of Appeals would mean that the local government units can no longer expropriate agricultural lands needed for the
construction of roads, bridges, schools, hospitals, etc, without first applying for conversion of the use of the lands with the Department of
Agrarian Reform, because all of these projects would naturally involve a change in the land use. In effect, it would then be the Department
of Agrarian Reform to scrutinize whether the expropriation is for a public purpose or public use.

Ordinarily, it is the legislative branch of the local government unit that shall determine whether the use of the property sought to be
expropriated shall be public, the same being an expression of legislative policy. The courts defer to such legislative determination and
will intervene only when a particular undertaking has no real or substantial relation to the public use.

The expropriation is for a public purpose, hence the resolution is authorized and valid.

When the Court of Appeals ordered the suspension of the proceedings until the Province of Camarines Sur shall have obtained the
authority of the Department of Agrarian Reform to change the classification of the lands sought to be expropriated from agricultural to
non-agricultural use, it assumed that the resolution is valid and that the expropriation is for a public purpose or public use.

There has been a shift from the literal to a broader interpretation of "public purpose" or "public use" for which the power of eminent domain
may be exercised. The old concept was that the condemned property must actually be used by the general public (e.g. roads, bridges,
public plazas, etc.) before the taking thereof could satisfy the constitutional requirement of "public use". Under the new concept, "public
use" means public advantage, convenience or benefit, which tends to contribute to the general welfare and the prosperity of the whole
community, like a resort complex for tourists or housing project.

The expropriation of the property authorized by the questioned resolution is for a public purpose. The establishment of a pilot development
center would inure to the direct benefit and advantage of the people of the Province of Camarines Sur. Once operational, the center would
make available to the community invaluable information and technology on agriculture, fishery and the cottage industry. Ultimately, the
livelihood of the farmers, fishermen and craftsmen would be enhanced. The housing project also satisfies the public purpose requirement
of the Constitution. As held in Sumulong v. Guerrero, 154 SCRA 461, "Housing is a basic human need. Shortage in housing is a matter
of state concern since it directly and significantly affects public health, safety, the environment and in sum the general welfare."
---------------------------------------------------XXX--------------------------------------------
BRGY. SAN ROQUE v. HEIRS OF PASTOR
G.R. No. 138896 June 20, 2000

FACTS: In 1997, Brgy. San Roque of Talisay, Cebu filed for an expropriation suit before the MTC of Talisay against the heirs of Franco
Pastor. The MTC denied the suit because apparently under BP 129, MTCs do not have jurisdiction over expropriation cases as it is the
RTCs that are lodged with the power to try such cases. So Brgy. San Roque filed it before RTC Talisay but then Judge Jose Soberano,
Jr. denied the suit as he ruled that the action for eminent domain affected title to real property; hence, the value of the property to be
expropriated would determine whether the case should be filed before the MTC or the RTC. The judge also concluded that the action
should have been filed before the MTC since the value of the subject property was less than P20,000.

ISSUE: Whether or not the RTC should take cognizance of the expropriation case.

HELD: Yes. Under Section 19 (1) of BP 129, which provides that RTCs shall exercise exclusive original jurisdiction over “all civil actions
in which the subject of the litigation is incapable of pecuniary estimation; . . . . .” The present action involves the exercise of the right to
eminent domain, and that such right is incapable of pecuniary estimation.

What are the two phases of expropriation cases?

The first is concerned with the determination of the authority of the plaintiff to exercise the power of eminent domain and the propriety of
its exercise in the context of the facts involved in the suit. It ends with an order, if not of dismissal of the action, “of condemnation declaring
that the plaintiff has a lawful right to take the property sought to be condemned, for the public use or purpose described in the complaint,
upon the payment of just compensation to be determined as of the date of the filing of the complaint.” An order of dismissal, if this be
ordained, would be a final one, of course, since it finally disposes of the action and leaves nothing more to be done by the Court on the
merits. So, too, would an order of condemnation be a final one, for thereafter as the Rules expressly state, in the proceedings before the
Trial Court, “no objection to the exercise of the right of condemnation (or the propriety thereof) shall be filed or heard.”

The second phase of the eminent domain action is concerned with the determination by the court of “the just compensation for the
property sought to be taken.” This is done by the Court with the assistance of not more than three (3) commissioners. The order fixing
the just compensation on the basis of the evidence before, and findings of, the commissioners would be final, too. It would finally dispose
of the second stage of the suit, and leave nothing more to be done by the Court regarding the issue. . . .

It should be stressed that the primary consideration in an expropriation suit is whether the government or any of its instrumentalities has
complied with the requisites for the taking of private property. Hence, the courts determine the authority of the government entity, the
necessity of the expropriation, and the observance of due process. In the main, the subject of an expropriation suit is the government’s
exercise of eminent domain, a matter that is incapable of pecuniary estimation.
-----------------------------------------------XXX---------------------------------------------
MUNICIPALITY OF PARANAQUE V VM REALTY
G.R. NO. 127820. JULY 20, 1998

Facts:
Under a city council resolution, the Municipality of Parañaque filed on September 20, 1993, a Complaint for expropriation against Private
Respondent V.M. Realty Corporation over two parcels of land of 10,000 square meters. The city previously negotiated for the sale of the
property but VM didn’t accept.
The trial court issued an Order dated February 4, 1994, authorizing petitioner to take possession of the subject property upon deposit
with its clerk of court of an amount equivalent to 15 percent of its fair market value based on its current tax declaration.
According to the respondent, the complaint failed to state a cause of action because it was filed pursuant to a resolution and not to an
ordinance as required by RA 7160 (the Local Government Code); and (b) the cause of action, if any, was barred by a prior judgment or
res judicata. Petitioner claimed that res judicata was not applicable.
The trial court dismissed the case. The petitioner’s MFR was denied. The CA affirmed.

Issues:
1. WON a resolution duly approved by the municipal council has the same force and effect of an ordinance and will not deprive an
expropriation case of a valid cause of action.
2. WON the principle of res judicata as a ground for dismissal of case is not applicable when public interest is primarily involved.

Held: No to 1st Yes to 2nd. Petition dismissed.

Ratio:
1. Petitioner contends that a resolution approved by the municipal council for the purpose of initiating an expropriation case “substantially
complies with the requirements of the law” because the terms “ordinance” and “resolution” are synonymous for “the purpose of bestowing
authority [on] the local government unit through its chief executive to initiate the expropriation proceedings in court in the exercise of the
power of eminent domain.
To strengthen this point, the petitioner cited Article 36, Rule VI of the Rules and Regulations Implementing the Local Government Code,
which provides: “If the LGU fails to acquire a private property for public use, purpose, or welfare through purchase, the LGU may
expropriate said property through a resolution of the Sanggunian authorizing its chief executive to initiate expropriation proceedings.”
Court-No. The power of eminent domain is lodged in the legislative branch of government, which may delegate the exercise thereof to
LGUs, other public entities and public utilities. An LGU may therefore exercise the power to expropriate private property only when
authorized by Congress and subject to the latter’s control and restraints, imposed “through the law conferring the power or in other
legislations.

Sec 19, RA 7160 - A local government unit may, through its chief executive and acting pursuant to an ordinance, exercise the power of
eminent domain for public use, or purpose, or welfare for the benefit of the poor and the landless, upon payment of just compensation,
pursuant to the provisions of the Constitution and pertinent laws.

Thus, the following essential requisites must concur before an LGU can exercise the power of eminent domain:
1. An ordinance is enacted by the local legislative council authorizing the local chief executive, in behalf of the LGU, to exercise the
power of eminent domain or pursue expropriation proceedings over a particular private property.
2. The power of eminent domain is exercised for public use, purpose or welfare, or for the benefit of the poor and the landless.
3. There is payment of just compensation, as required under Section 9, Article III of the Constitution, and other pertinent laws.
4. A valid and definite offer has been previously made to the owner of the property sought to be expropriated, but said offer was not
accepted.

In the case at bar, the local chief executive sought to exercise the power of eminent domain pursuant to a resolution of the municipal
council. Thus, there was no compliance with the first requisite that the mayor be authorized through an ordinance.

We are not convinced by petitioner’s insistence that the terms “resolution” and “ordinance” are synonymous. A municipal ordinance is
different from a resolution. An ordinance is a law, but a resolution is merely a declaration of the sentiment or opinion of a lawmaking body
on a specific matter. An ordinance possesses a general and permanent character, but a resolution is temporary in nature.
If Congress intended to allow LGUs to exercise eminent domain through a mere resolution, it would have simply adopted the language
of the previous Local Government Code. But Congress did not. In a clear divergence from the previous Local Government Code, Section
19 of RA 7160 categorically requires that the local chief executive act pursuant to an ordinance. Moreover, the power of eminent domain
necessarily involves a derogation of a fundamental or private right of the people.[35] Accordingly, the manifest change in the legislative
language -- from “resolution” under BP 337 to “ordinance” under RA 7160 -- demands a strict construction.

When the legislature interferes with that right and, for greater public purposes, appropriates the land of an individual without his consent,
the plain meaning of the law should not be enlarged by doubtful interpretation.
Petitioner relies on Article 36, Rule VI of the Implementing Rules, which requires only a resolution to authorize an LGU to exercise eminent
domain. It is axiomatic that the clear letter of the law is controlling and cannot be amended by a mere administrative rule issued for its
implementation.
Strictly speaking, the power of eminent domain delegated to an LGU is in reality not eminent but “inferior” domain, since it must conform
to the limits imposed by the delegation, and thus partakes only of a share in eminent domain.

2. As correctly found by the Court of Appeals and the trial court, all the requisites for the application of res judicata are present in this
case. There is a previous final judgment on the merits in a prior expropriation case involving identical interests, subject matter and cause
of action, which has been rendered by a court having jurisdiction over it.
Be that as it may, the Court holds that the principle of res judicata, which finds application in generally all cases and proceedings, cannot
bar the right of the State or its agent to expropriate private property.
Eminent Domain can reach every form of property which the State might need for public use whenever they need it.
While the principle of res judicata does not denigrate the right of the State to exercise eminent domain, it does apply to specific issues
decided in a previous case.
In Republic vs De Knecht, the Court ruled that the power of the State or its agent to exercise eminent domain is not diminished by the
mere fact that a prior final judgment over the property to be expropriated has become the law of the case as to the parties. The State or
its authorized agent may still subsequently exercise its right to expropriate the same property, once all legal requirements are complied
with.
---------------------------------------------XXX--------------------------------------------
CITY OF CEBU v. THE COURT OF APPEALS

One of the basic tenets of procedural law is a liberal interpretation of the Rules of Court in order to promote their object and to assist the
parties in obtaining just, speedy, and inexpensive determination of every action and proceeding. Time and again, this Court has stressed
that the primordial concern of rules of procedure is to secure substantial justice. Otherwise stated, they are but a means to an end. Hence,
a rigid and technical enforcement of these rules which overrides the ends of justice shall not be countenanced. Substance cannot be
subordinated to procedure when to do so would deprive a party of his day in court on the basis solely of a technicality. The case before
us illustrates how a stringent application of procedural rules, when uncalled for, can result in a contravention of the foregoing principle
and the consequent subversion of justice.

Facts: Cardeno is the owner of a parcel of land located at Sitio Sto. Nino, Alaska-Mambaling, covered by TCT No. 116692. Petitioner,
City of Cebu, filed a complaint for eminent domain against private respondent with the RTC of Cebu City seeking to expropriate the said
parcel of land. The complaint was initiated pursuant to Resolution No. 404 and Ordinance No.1418, dated February 17, 1992, of the
Sangguniang Panlungsod of Cebu City authorizing the City Mayor to expropriate the said parcel of land for the purpose of providing a
socialized housing project for the landless and low-income city residents. Private respondent filed a motion to dismiss the said complaint
on the ground of lack of cause of action. She argued that the allegations contained in paragraph VII of the complaint, to wit: “That repeated
negotiations had been made with the defendant to have the aforementioned property purchased by the plaintiff through negotiated sale
without resorting to expropriation, but said negotiations failed.” do not show compliance with one of the conditions precedent to the
exercise of the power of eminent domain by a local government unit as enunciated in Section 19 of R.A. 7160 which provides in part that:
“A local government unit may, through its chief executive and acting pursuant to an ordinance, exercise the power of eminent domain x
x x; Provided, however, That the power of eminent domain may not be exercised unless a valid and definite offer has been previously
made to the owner, and such offer was not accepted x x x .” Petitioner sought to establish compliance with the abovecited requirement
by alleging in its “Comment and Opposition” the following facts: “7. To further pursue its desire to acquire the property concerned, the
plaintiff made on October 28,1991, another offer to Mrs. Cardeno, through her lawyer, Atty. Omar Redula, for the purchase of her property
in the amount of Four Hundred Seventy Eight Thousand (P478,000.00) Pesos. x x x; “8. The said offer was again refused, thus the resort
by the plaintiff to expropriation.”

RTC Decision: dismissed the complaint and ruled as follows: “The import or meaning of the said allegation in paragraph VII of the
complaint . . . does not convey or connote the same meaning or import or even approximate, the condition precedent required which is .
. .‘Unless a valid and definite offer has been previously made to the owner and such offer was not accepted.’ "The Court is of the opinion
that the City of Cebu has not complied with the condition precedent, hence, the complaint does not state a cause of action.” Furthermore,
. . .the RTC invoked the oft-cited rule that where the ground for dismissal is that the complaint states no cause of action, its sufficiency
can only be
determined from the facts alleged in the complaint and no other.

CA Decision: affirmed the ruling of the RTC. According to the CA, an allegation of repeated negotiations made with the private respondent
for the purchase of her property by the petitioner, “cannot by any stretch of imagination, be equated or likened to the clear and specific
requirement that the petitioner should have previously made a valid and definite offer to purchase.” It further added that the term
“negotiation” which necessarily implies uncertainty, it consisting of acts the purpose of which is to arrive at a conclusion, may not be
perceived to mean the valid and definite offer contemplated by law. Hence this petition.

Issues:
1. WON the complaint states a cause of action. YES
2. WON the City of Cebu had the right to expropriate the subject parcel of land.

RULING
1. While petitioner reiterates that paragraph VII of the complaint sufficiently states compliance with the requirement of “a valid and definite
offer,” private respondent insists that the term “negotiations” is too broad to be equated with the said requirement. Private respondent
posited that by definition, “negotiations run the whole range of acts preparatory to concluding an agreement, from the preliminary
correspondence; the fixing of the terms of the agreement; the price; the mode of payment; obligations of (sic) the parties may conceive
as necessary to their agreement.” Thus, “negotiations” by itself may pertain to any of the foregoing and does not automatically mean the
making of “a valid and definite offer.” or uncertain but, nevertheless, a cause of action can, in any manner, be made out therefrom, and
the plaintiff would be entitled to recover in any aspect of the facts or any combination of the facts alleged, if they were to be proved, then
the motion to dismiss should be denied.” In other words, a complaint should not be dismissed for insufficiency unless it appears clearly
from the face of the complaint that the plaintiff is not entitled to any relief under any state of facts which could be proved within the facts
alleged therein. The error of both the RTC and respondent CA in holding that the complaint failed to state a cause of action stems from
their inflexible application of the rule that: when the motion to dismiss is based on the ground that the complaint states no cause of action,
no evidence may be allowed and the issue should only be determined in the light of the allegations of the complaint. However, this rule
is not without exceptions. . . .The same liberality (as the cases mentioned here) should be applied in the instant case where an examination
of petitioner’s “Comment and Opposition” to private respondent’s Motion to Dismiss leaves no room for doubt that petitioner had indeed
made “a valid and definite offer” to private respondent as required by law.

2. Yes, City of Cebu had the right to expropriate. "x x x. Under R.A. 7160, Sec. 9 thereof, the City of Cebu is legally vested with the power
of eminent domain and pursuant thereto is filing this petition/complaint as authorized by Ordinance No.1418 passed by the Sangguniang
Panlungsod on February 17,1992, a photocopy of which is herein attached as Annex “A”, and made an integral part of this complaint. x
x x” [Italics supplied.] All documents attached to a complaint, the due execution and genuineness of which are not denied under oath by
the defendant, must be considered.

This Court finds that the complaint does in fact state a cause of action. . . A complaint should not be dismissed upon a mere ambiguity,
indefiniteness or uncertainty of the cause of action stated therein for these are not grounds for a motion to dismiss but rather for a bill of
particulars. And, “x x x though the allegations in the complaint are ambiguous, indefinite as part of the complaint without need of
introducing evidence thereon. Additionally, the general rule is that a motion to dismiss hypothetically admits the truth of the facts alleged
in the complaint.

Thus, Ordinance No.1418, with all its provisions, is not only incorporated into the complaint for eminent domain filed by petitioner, but is
also deemed admitted by private respondent. A perusal of the copy of said ordinance which has been annexed to the complaint shows
that the fact of petitioner’s having made a previous valid and definite offer to private respondent is categorically stated therein. Thus, the
second whereas clause of the said ordinance provides as follows: “WHEREAS, the city government has made a valid and definite offer
to purchase subject lot(s) for the public use aforementioned but the registered owner Mrs. Merlita Cardeno has rejected such offer.”

The foregoing should now put to rest the long drawn argument over the alleged failure of the complaint to state a cause of action. There
is no longer any room for doubt that as alleged in the complaint, and as admitted by private respondent, the petitioner had in fact complied
with the condition precedent of “a valid and definite offer” set forth in Sec. 19 of R.A. 7160. And as a fitting finale to this controversy, the
principle enunciated in both the Tan and Marcopper cases is here reiterated: “The rules of procedure are not to be applied in a very rigid,
technical sense; rules of procedure are used only to help secure substantial justice. If a technical and rigid enforcement of the rules is
made their aim would be defeated. Where the rules are merely secondary in importance are made to override the ends of justice; the
technical rules had been misapplied to the prejudice of the substantial right of a party, said rigid application cannot be countenanced.”

The aforequoted doctrine finds compelling application in the case at bench. For as correctly averred by petitioner, nothing else was
acomplished by the dismissal of the complaint for eminent domain but a considerable delay in the proceedings. The dismissal of the
complaint did not bar petitioner from filing another eminent domain case and from correcting its alleged error by the mere expedient of
changing paragraph VII thereof. Indeed, precious time has been wasted while the salutary objectives of Ordinance No. 1418 of the City
of Cebu have been put on hold by a quarrel over technical matters.

WHEREFORE, the petition is hereby GRANTED and the decision appealed from is REVERSED and SET ASIDE. The case is ordered
remanded to the RTC which shall proceed to the hearing and final determination thereof.
---------------------------------------------XXX-----------------------------------------
HEIRS OF JUANCHO ARDONA V. REYES
123 SCRA 220

Facts:

The Philippine Tourism Authority filed 4 complaints with the CFI of Cebu City for the expropriation of 282 ha of rolling land situated in
barangays Malubog and Babag, Cebu City for the development into integrated resort complexes of selected and well-defined geographic
areas with potential tourism value. The PTA will construct a sports complex, club house, golf course, playground and picnic area on said
land. An electric power grid will also be established by NPC as well as deep well and drainage system. Complimentary support facilities
(malls, coffee shops, etc) will also be created. The defendants alleged that the taking is allegedly not impressed with public use under
the Constitution. Also, assuming that PTA has such power, the intended use cannot be paramount to the determination of the land as a
land reform area; that limiting the amount of compensation by legislative fiat is constitutionally repugnant; and that since the land is under
the land reform program, it is the Court of Agrarian Relations and not the Court of First Instance, that has jurisdiction over the expropriation
cases. The Philippine Tourism Authority having deposited with the PNB, an amount equivalent to 10% of the value of the properties
pursuant to PD1533, the lower court issued separate orders authorizing PTA to take immediate possession of the premises and directing
the issuance of writs of possession. The petitioners who are occupants of the lands, filed a petition for certiorari in the SC. They contended
that (1) the taking was not for public use; (2) the land was covered by the land reform program; and (3) expropriation would impair the
obligation of contracts.

Issue: WON the public use requirement has been complied with

Held:

Yes, There are three provisions of the Constitution which directly provide for the exercise of the power of eminent domain. Sec 2, Article
IV states that private property shall not be taken for public use without just compensation. Section 6, Article XIV allows the State, in the
interest of national welfare or defense and upon payment of just compensation to transfer to public ownership, utilities and other private
enterprises to be operated by the government. Section 13, Article XIV states that the Batasang Pambansa may authorize upon payment
of just compensation the expropriation of private lands to be subdivided into small lots and conveyed at cost to deserving citizens.

The concept of public use is not limited to traditional purposes for the construction of roads, bridges, and the like. The idea that "public
use" means "use by the public" has been discarded. As long as the purpose of the taking is public, then the power of eminent domain
comes into play. It is accurate to state then that at present whatever may be beneficially employed for the general welfare satisfies the
requirement of public use. The petititioners have not shown that the area being developed is land reform area and that the affected
persons have been given emancipation patents and certificates of land transfer. The contract clause has never been regarded as a barrier
to the exercise of the police power and likewise eminent domain.

Below is a long explanation I got from the net:

Held: Yes Ratio: There are three provisions of the Constitution which directly provide for the exercise of the power of eminent domain.
Sec 2, Article IV states that private property shall not be taken for public use without just compensation. Section 6, Article XIV allows the
State, in the interest of national welfare or defense and upon payment of just compensation to transfer to public ownership, utilities and
other private enterprises to be operated by the government. Section 13, Article XIV states that the Batasang Pambansa may authorize
upon payment of just compensation the expropriation of private lands to be subdivided into small lots and conveyed at cost to deserving
citizens. While not directly mentioning the expropriation of private properties upon payment of just compensation, the provisions on social
justice and agrarian reforms which allow the exercise of police power together with the power of eminent domain in the implementation
of constitutional objectives are even more far reaching insofar as taxing of private property is concerned. We cite all the above provisions
on the power to expropriate because of the petitioners' insistence on a restrictive view of the eminent domain provision.

The thrust of all constitutional provisions on expropriation is in the opposite direction. As early as 1919, this Court in Visayan Refining Co.
v. Samus categorized the restrictive view as wholly erroneous and based on a misconception of fundamentals. The petitioners look for
the word "tourism" in the Constitution. Understandably the search would be in vain. To freeze specific programs like tourism into express
constitutional provisions would make the Constitution more prolix than a bulky code and require of the framers a prescience beyond
Delphic proportions. In said case, this Court emphasized that the power of eminent domain is inseparable from sovereignty being essential
to the existence of the State and inherent in government even in its most primitive forms. The only purpose of the provision in the Bill of
Rights is to provide some form of restraint on the sovereign power. It is not a grant of authority . The petitioners ask us to adopt a strict
construction and declare that "public use" means literally use by the public and that "public use" is not synonymous with "public interest",
"public benefit", or "public welfare" and much less "public convenience." The petitioners face two major obstacles. First, their contention
which is rather sweeping in its call for a retreat from the public welfare orientation is unduly restrictive and outmoded. Second, no less
than the lawmaker has made a policy determination that the power of eminent domain may be exercised in the promotion and
development of Philippine tourism.

The restrictive view of public use may be appropriate for a nation which circumscribes the scope of government activities and public
concerns and which possesses big and correctly located public lands that obviate the need to take private property for public purposes.
Neither circumstance applies to the Philippines. We have never been a laissez faire State. And the necessities which impel the exertion
of sovereign power are all too often found in areas of scarce public land or limited government resources.There can be no dou bt that
expropriation for such traditional purposes as the construction of roads, bridges, ports, waterworks, schools, electric and
telecommunications systems, hydroelectric power plants, markets and slaughterhouses, parks, hospitals, government office buildings,
and flood control systems is valid. However, the concept of public use is not limited to traditional purposes. Here as elsewhere the idea
that "public use" is strictly limited to clear cases of "use by the public" has been discarded. In the Philippines, Chief Justice Enrique M.
Fernando has aptly summarized the statutory and judicial trend as follows: "The taking to be valid must be for public use. There was a
time when it was felt that a literal meaning should be attached to such a requirement. Whatever project is undertaken must be for the
public to enjoy, as in the case of streets or parks. Otherwise, expropriation is not allowable. It is not any more. As long as the purpose of
the taking is public, then the power of eminent domain comes into play. As just noted, the constitution in at least two cases, to remove
any doubt, determines what is public use. One is the expropriation of lands to be subdivided into small lots for resale at cost to individuals.
The other is in the transfer, through the exercise of this power, of utilities and other private enterprise to the government. It is accurate to
state then that at present whatever may be beneficially employed for the general welfare satisfies the requirement of public use."

The petitioners' contention that the promotion of tourism is not "public use" because private concessioners would be allowed to maintain
various facilities such as restaurants, hotels, stores, etc. inside the tourist complex is impressed with even less merit. Private bus firms,
taxicab fleets, roadside restaurants, and other private businesses using public streets and highways do not diminish in the least bit the
public character of expropriations for roads and streets. The lease of store spaces in underpasses of streets built on expropriated land
does not make the taking for a private purpose. Airports and piers catering exclusively to private airlines and shipping companies are still
for public use. The expropriation of private land for slum clearance and urban development is for a public purpose even if the developed
area is later sold to private homeowners, commercial firms, entertainment and service companies, and other private concerns.

The petitioners have also failed to overcome the deference that is appropriately accorded to formulations of national policy expressed in
legislation. The rule in Berman v. Parker (supra) of deference to legislative policy even if such policy might mean taking from one private
person and conferring on another private person applies as well as in the Philippines. An examination of the language in the 1919 cases
of City of Manila v. Chinese Community of Manila and Visayan Refining Co. v. Camus, earlier cited, shows that from the very start of
constitutional government in our country judicial deference to legislative policy has been clear and manifest in eminent domain
proceedings.

The expressions of national policy are found in the revised charter of the Philippine Tourism Authority, PD 564. (Disregard of Land Reform
Nature) According to them, assuming that PTA has the right to expropriate, the properties subject of expropriation may not be taken for
the purposes intended since they are within the coverage of "operation land transfer" under the land reform program; that the agrarian
reform program occupies a higher level in the order of priorities than other State policies like those relating to the health and physical
well-being of the people; and that property already taken for public use may not be taken for another public use. The petitioners, however,
have failed to show that the area being developed is indeed a land reform area and that the affected persons have emancipation patents
and certificates of land transfer. The records show that the area being developed into a tourism complex consists of more than 808
hectares, almost all of which is not affected by the land reform program. The portion being expropriated is 282 hectares of hilly and
unproductive land where even subsistence farming of crops other than rice and corn can hardly survive. And of the 282 disputed hectares,
only 8,970 square meters - less than one hectare - is affected by Operation Land Transfer. Of the 40 defendants, only two have
emancipation patents for the less than one hectare of land affected. (Non Impairment Clause) The non-impairment clause has never
been a barrier to the exercise of police power and likewise eminent domain. As stated in Manigault v. Springs "parties by entering into
contracts may not estop the legislature from enacting laws intended for the public good." The applicable doctrine is expressed in Arce v.
Genato which involved the expropriation of land for a public plaza.

The issue of prematurity is also raised by the petitioners. They claim that since the necessity for the taking has not been previously
established, the issuance of the orders authorizing the PTA to take immediate possession of the premises, as well as the corresponding
writs of possession was premature. Under Presidential Decree No. 42, as amended by Presidential Decree No. 1533, the government,
its agency or instrumentality, as plaintiff in an expropriation proceedings is authorized to take immediate possession, control and
disposition of the property and the improvements, with power of demolition, notwithstanding the pendency of the issues before the court,
upon deposit with the Philippine National Bank of an amount equivalent to 10% of the value of the property expropriated.

The issue of immediate possession has been settled in Arce v. Genato. In answer to the issue: ". . . condemnation or expropriation
proceedings is in the nature of one that is quasi-in-rem, wherein the fact that the owner of the property is made a party is not essentially
indispensable insofar at least as it concerns the immediate taking of possession of the property and the preliminary determination of its
value, including the amount to be deposited."

Makasiar: It appearing that the petitioners are not tenants of the parcels of land in question and therefore do not fall within the purview of
the Land Reform Code, the petition should be dismissed on that score alone. There is no need to decide whether the power of the PTA
to expropriate the land in question predicated on the police power of the State shall take precedence over the social justice guarantee in
favor of tenants and the landless. The welfare of the landless and small land owners should prevail over the right of the PTA to expropriate
the lands just to develop tourism industry, which benefit the wealthy only. Such a position would increase the disenchanted citizens and
drive them to dissidence. The government is instituted primarily for the welfare of the governed and there are more poor people in this
country than the rich. The tourism industry is not essential to the existence of the government, but the citizens are, and their right to live
in dignity should take precedence over the development of the tourism industry.
---------------------------------------------------XXX--------------------------------------------
FILSTREAM INTERNATIONAL INC v. CA

FACTS:

Filstream International is the registered owner of parcels of land located in Antonio Rivera St., Tondo II Manila. On January 7, 1993, it
filed an ejectment suit against the occupants (private respondents) of the said parcels of land on the grounds of termination of the lease
contract and non--payment of rentals. The ejectment suit became final and executory as no further action was taken beyond the CA.

During the pendency of the ejectment proceedings private respondents filed a complaint for Annulment of Deed of Exchange against
Filstream. The City of Manila came into the picture when it approved Ordinance No. 7813 authorizing Mayor Alfredo Lim to initiate
acquisition through legal means of certain parcels of land. Subsequently, the City of Manila approved Ordinance No. 7855 declaring the
expropriation of certain parcels of land which formed part of the properties of Filstream. The said properties were sold and distributed to
qualified tenants pursuant to the Land Use Development Program of the City of Manila. The City of Manila then filed a complaint for
eminent domain seeking to expropriate lands in Antonio Rivera St. The RTC issued a Writ of Possession in favor of the City.

Filstream filed a motion to dismiss and a motion to quash the writ of possession. The motion to dismiss was premised on the following
grounds: no valid cause of action; the petition does not satisfy the requirements of public use and a mere clandestine maneuver to
circumvent the writ execution issued by the RTC of Manila in the ejectment suit; violation of the constitutional guarantee against non-
impairment of obligation and contract; price offered was too low hence violative of the just compensation provision of the constitution.

The RTC denied the two motions. Filstream filed a Petition for Certiorari with the CA which dismissed the petition for being insufficient in
form and substance, aside from the fact that copies of the pleadings attached to the petition are blurred and unreadable.

ISSUES/HELD:

1. WON City of Manila may exercise right of eminent domain despite the existence of a final and executory judgment ordering private
respondents to vacate the lots.

YES. Petitioner Filstream anchors its claim by virtue of its ownership over the properties and the existence of a final and executory
judgment against private respondents ordering the latter’s ejectment from the premises.

Private respondents’ claim on the other hand hinges on an alleged supervening event which has rendered the enforcement of petitioner’s
rights moot, that is, the expropriation proceedings undertaken by the City of Manila over the disputed premises for the benefit of herein
private respondents. For its part, the City of Manila is merely exercising its power of eminent domain within its jurisdiction by expropriating
petitioner’s properties for public use.

There is no dispute as to the existence of a final and executory judgment in favor of petitioner Filstream ordering the ejectment of private
respondents from the properties subject of this dispute. Thus, petitioner has every right to assert the execution of this decision as it had
already became final and executory.

However, it must also be conceded that the City of Manila has an undeniable right to exercise its power of eminent domain within its
jurisdiction. The right to expropriate private property for public use is expressly granted to it under Sec 19 of the Local Government Code.
Sec 100 of the Revised Charter of the City of Manila further empowers the city government to expropriate private property in the pursuit
of its urban land reform and housing program. The city’s right to exercise these prerogatives notwithstanding the existence of a final and
executory judgment over the property to be expropriated had already been previously upheld by the court in the case of Philippine
Columbian Association vs Panis:

“The City of Manila, acting through its legislative branch, has the express power to acquire private lands in the city and subdivide these
lands into home lots for sale to bona-fide tenants or occupants thereof, and to laborers and low-salaried employees of the city.

That only a few could actually benefit from the expropriation of the property does not diminish its public use character. It is simply not
possible to provide all at once land and shelter for all who need them (Sumulong v. Guerrero, 154 SCRA 461 [1987]).

Corollary to the expanded notion of public use, expropriation is not anymore confined to vast tracts of land and landed estates. It is
therefore of no moment that the land sought to be expropriated in this case is less than the half a hectare only (Pulido v. Court of Appeals,
122 SCRA 63 [1983]).

2. WON expropriation of Filstream’s lots were legally and validly undertaken.

NO. We take judicial notice of the fact that urban land reform has become a paramount task in view of the acute shortage of decent
housing in urban areas particularly in Metro Manila. Nevertheless, despite the existence of a serious dilemma, local government units
are not given an unbridled authority when exercising their power of eminent domain in pursuit of solutions to these problems.
Constitutional provisions on due process and just compensation for the expropriation of private property must be compli ed with. Other
laws have also set down specific rules in the exercise of the power of eminent domain, to wit:

• Sec 19 of LGC provides that such exercise must be pursuant to the provisions of the Constitution and pertinent laws.

• Sec 9 of the Urban Development and Housing Act of 1992 (UDHA) provides an order of priority in the acquisition of land for socialized
housing, with private lands listed as the last option.

• Sec 10 of UDHA provides that expropriation shall be resorted to only when other modes of acquisition such as community mortgage,
land swapping, donation to the government, etc. have been exhausted, and, where expropriation is resorted to, parcels of land owned by
small property owners shall be exempted.

Compliance with the above legislated conditions are deemed mandatory because these are the only safeguards in securing the right of
owners of private property to DUE PROCESS when their property is expropriated for public use.

There is nothing in the records which would indicate that the City of Manila complied with the above conditions. Filstream’s properties
were expropriated and ordered condemned in favor of the City of Manila sans any showing that resort to the acquisition of other lands
listed under Sec. 9 of RA 7279 have proved futile. Evidently, there was a violation of petitioner Filstream’s right to due process.

It must be emphasized that the State has a paramount interest in exercising its power of eminent domain for the general good considering
that the right of the State to expropriate private property as long as it is for public use always takes precedence over the interest of private
property owners. However we must not lose sight of the fact that the individual rights affected by the exercise of such right are also
entitled to protection, bearing in mind that the exercise of this superior right cannot override the guarantee of due process extended by
the law to owners of the property to be expropriated.
----------------------------------------XXX--------------------------------------
HAGONOY MARKET VENDOR ASSOCIATION, petitioner, vs. MUNICIPALITY OF HAGONOY, BULACAN, respondent.
G.R. No. 137621. February 6, 2002
PUNO, J.
(Jeka)
Tickler: The Sanggunian Bayan of Hagonoy enacted an ordinace, Kautusan Blg. 28, increasing the stall rentals of the market vendors in
Hagonoy. Article 3 provided that it shall take effect upon approval. The petitioner’s President filed an appeal with the Secretary of Justice
assailing the constitutionality of the tax ordinance. The Secretary of Justice dismissed the appeal on the ground that it was filed out f time
(beyond 30 days from the effectivity of the Ordinance).
SC held that the appeal of the petitioner with the Secretary of Justice is already time-barred. The periods stated in Section 187 of the
Local Government Code are mandatory. Ordinance No. 28 is a revenue measure adopted by the municipality of Hagonoy to fix and
collect public market stall rentals. It is essential that the validity of revenue measures is not left uncertain for a considerable length of time.
Furthermore, it was held that Section 6c.04 of the 1993 Municipal Revenue Code and Section 191 of the Local Government Code limiting
the percentage of increase that can be imposed apply to tax rates, not rentals.
Facts:
1. The Sangguniang Bayan of Hagonoy, Bulacan, enacted an ordinance,
a. Kautusan Blg. 28: increasing the stall rentals of the market vendors in Hagonoy.
b. Article 3 provided that it shall take effect upon approval.
c. The subject ordinance was posted from November 4-25, 1996.
2. The petitioner’s members were personally given copies of the approved Ordinance and were informed that it shall be enforced
in January, 1998.
a. The petitioner’s President filed an appeal with the Secretary of Justice assailing the constitutionality of the tax
ordinance.
b. Petitioner claimed it was unaware of the posting of the ordinance.
3. Respondent opposed the appeal.
a. It contended that the ordinance took effect on October 6, 1996 and that the ordinance, as approved, was
posted as required by law.
4. Petitioner’s appeal, made over a year later, was already time-barred.
a. Secretary of Justice: Dismissed the appeal on the ground that it was filed out of time, i.e., beyond thirty (30)
days from the effectivity of the Ordinance on October 1, 1996
b. Citing the case of Tañada vs. Tuvera, the Secretary of Justice held that the date of effectivity of the subject ordinance
retroacted to the date of its approval in October 1996, after the required publication or posting has been complied with,
pursuant to Section 3 of said ordinance
5. Motion for reconsideration was denied
a. Petitioner appealed to the Court of Appeals.
b. Petitioner did not assail the finding of the Secretary of Justice that their appeal was filed beyond the
reglementary period.
c. Instead, it urged that the Secretary of Justice should have overlooked this “mere technicality” and ruled on its petition
on the merits
6. CA: dismissed for being formally deficient as it was not accompanied by certified true copies of the assailed Resolutions of the
Secretary of Justice
Issue: WON the appeal by the petitioner with the Secretary of Justice time-barred.
Held: YES. The appeal of the petitioner with the Secretary of Justice is already time-barred.
Ratio:
1. On failure to attach certified true copies of the assaild decision of the Secretary of Justice:
a. the petitioner satisfactorily explained the circumstances relative to its failure to attach to its appeal certified true copies
of the assailed Resolutions of the Secretary of Justice
b. Due to bad weather, the person in charge (at the Department of Justice) was no longer available to certify to
(sic) the Resolutions.
c. The following day, October 22, 1998, was declared a non-working holiday because of (t)yphoon “Loleng.”
d. We find that the Court of Appeals erred in dismissing petitioner’s appeal on the ground that it was formally
deficient.
e. It is clear from the records that the petitioner exerted due diligence to get the copies of its appealed Resolutions certified
by the Department of Justice, but failed to do so on account of typhoon “Loleng.”
f. respondent appellate court should have tempered its strict application of procedural rules in view of the fortuitous event
considering that litigation is not a game of technicalities
2. However, the appeal of the petitioner with the Secretary of Justice is already time-barred.
a. Any question on the constitutionality or legality of tax ordinances or revenue measures may be raised on
appeal within thirty (30) days from the effectivity thereof to the Secretary of Justice
b. Section 187 of the Local Gov’t Code : appeal of a tax ordinance or revenue measure should be made to the Secretary
of Justice within thirty (30) days from effectivity of the ordinance and even during its pendency, the effectivity of the
assailed ordinance shall not be suspended.
c. Municipal Ordinance No. 28 took effect in October 1996. Petitioner filed its appeal only in December 1997, more than
a year after the effectivity of the ordinance in 1996.
d. Clearly, the Secretary of Justice correctly dismissed it for being time-barred.
e. The timeframe fixed by law for parties to avail of their legal remedies before competent courts is not a “mere technicality”
that can be easily brushed aside.
3. The periods stated in Section 187 of the Local Government Code are mandatory
a. Ordinance No. 28 is a revenue measure adopted by the municipality of Hagonoy to fix and collect public market stall
rentals.
b. It is essential that the validity of revenue measures is not left uncertain for a considerable length of time.
4. On the argument that its period to appeal should be counted not from the time the ordinance took effect in 1996 but from the
time its members were personally given copies of the approved ordinance in November 1997.
a. Two (2) grounds: first, no public hearing was conducted prior to the passage of the ordinance and, second, the
approved ordinance was not posted.
b. Petitioner’s bold assertion that there was no public hearing conducted prior to the passage of Kautusan Blg.
28 is belied by its own evidence.
c. In petitioner’s two (2) communications with the Secretary of Justice, it enumerated the various objections raised by its
members before the passage of the ordinance in several meetings called by the Sanggunian for the purpose.
d. These show beyond doubt that petitioner was aware of the proposed increase and in fact participated in the public
hearings therefore.
5. On the issue of publication or posting
a. In contrast, the respondent Sangguniang Bayan of the Municipality of Hagonoy, Bulacan, presented evidence
which clearly shows that the procedure for the enactment of the assailed ordinance was complied with
b. After its approval, copies of the Ordinance were given to the Municipal Treasurer on the same day.
c. On November 9, 1996, the Ordinance was approved by the Sangguniang Panlalawigan. The Ordinance was
posted during the period from November 4 - 25, 1996 in three (3) public places, viz: in front of the municipal
building, at the bulletin board of the Sta. Ana Parish Church and on the front door of the Office of the Market
Master in the public market
d. Posting was validly made in lieu of publication as there was no newspaper of local circulation in the
municipality of Hagonoy.
6. Section 6c.04 of the 1993 Municipal Revenue Code and Section 191 of the Local Government Code limiting the
percentage of increase that can be imposed apply to tax rates, not rentals.
a. Neither can it be said that the rates were not uniformly imposed or that the public markets included in the Ordinance
were unreasonably determined or classified.
b. To be sure, the Ordinance covered the three (3) concrete public markets: the two-storey Bagong Palengke, the burnt
but reconstructed Lumang Palengke and the more recent Lumang Palengkewith wet market. However,
the Palengkeng Bagong Munisipyo or Gabaldon was excluded from the increase in rentals as it is only a makeshift,
dilapidated place, with no doors or protection for security, intended for transient peddlers who used to sell their goods
along the sidewalk
----------------------------------------------------------------XXX----------------------------------------------------
REPUBLIC vs. CA
G.R. No. 146587 July 2, 2002

FACTS:

Petitioner (PIA) instituted expropriation proceedings covering a total of 544,980 square meters of contiguous land situated along
MacArthur Highway, Malolos, Bulacan, to be utilized for the continued broadcast operation and use of radio transmitter facilities for the
“Voice of the Philippines” project.

Petitioner made a deposit of P517,558.80, the sum provisionally fixed as being the reasonable value of the property. On 26 February
1979, or more than 9 years after the institution of the expropriation proceedings, the trial court issued this order condemning the property
and ordering the plaintiff to pay the defendants the just compensation for the property.

It would appear that the National Government failed to pay the respondents the just compensation pursuant to the foregoing decision.
The respondents then filed a manifestation with a motion seeking payment for the expropriated property. In response, the court issued a
writ of execution for the implementation thereof.

Meanwhile, Pres. Estrada issued Proc. No. 22 transferring 20 hectares of the expropriated land to the Bulacan State University.

Despite the court’s order, the Santos heirs remained unpaid and no action was on their case until petitioner filed its manifestation and
motion to permit the deposit in court of the amount P4,664,000 by way of just compensation.

The Santos heirs submitted a counter-motion to adjust the compensation from P6/sq.m. as previously fixed to its current zonal value of
P5,000/sq.m. or to cause the return of the expropriated property.

The RTC Bulacan ruled in favor of the Santos heirs declaring its 26 February 1979 Decision to be unenforceable on the ground of
prescription in accordance with Sec. 6, Rule 39 of the 1964/1997 ROC which states that a final and executory judgment or order may be
executed on motion within 5 years from the date of its entry. RTC denied petitioner’s Motion to Permit Deposit and ordered the return of
the expropriated property to the heirs of Santos.

ISSUES:

1. WON the petitioner may appropriate the property


2. WON the respondents are entitled to the return of the property in question

HELD:

1. The right of eminent domain is usually understood to be an ultimate right of the sovereign power to appropriate any property within its
territorial sovereignty for a public purpose. Fundamental to the independent existence of a State, it requires no recognition by the
Constitution, whose provisions are taken as being merely confirmatory of its presence and as being regulatory, at most, in the due exercise
of the power. In the hands of the legislature, the power is inherent, its scope matching that of taxation, even that of police power itself, in
many respects. It reaches to every form of property the State needs for public use and, as an old case so puts it, all separate interests
of individuals in property are held under a tacit agreement or implied reservation vesting upon the sovereign the right to resume the
possession of the property whenever the public interest so requires it.

The ubiquitous character of eminent domain is manifest in the nature of the expropriation proceedings. Expropriation proceedings are not
adversarial in the conventional sense, for the condemning authority is not required to assert any conflicting interest in the property. Thus,
by filing the action, the condemnor in effect merely serves notice that it is taking title and possession of the property, and the defendant
asserts title or interest in the property, not to prove a right to possession, but to prove a right to compensation for the taking.

Obviously, however, the power is not without its limits: first, the taking must be for public use, and second, that just compensation must
be given to the private owner of the property. These twin proscriptions have their origin in the recognition of the necessity for achieving
balance between the State interests, on the one hand, and private rights, upon the other hand, by effectively restraining the former and
affording protection to the latter. In determining “public use,” two approaches are utilized - the first is public employment or the actual use
by the public, and the second is public advantage or benefit. It is also useful to view the matter as being subject to constant growth, which
is to say that as society advances, its demands upon the individual so increases, and each demand is a new use to which the resources
of the individual may be devoted.

The expropriated property has been shown to be for the continued utilization by the PIA, a significant portion thereof being ceded for the
expansion of the facilities of the Bulacan State University and for the propagation of the Philippine carabao, themselves in line with the
requirements of public purpose. Respondents question the public nature of the utilization by petitioner of the condemned property, pointing
out that its present use differs from the purpose originally contemplated in the 1969 expropriation proceedings. The argument is of no
moment. The property has assumed a public character upon its expropriation. Surely, petitioner, as the condemnor and as the owner of
the property, is well within its rights to alter and decide the use of that property, the only limitation being that it be fo r public use, which,
decidedly, it is.

2. NO. In insisting on the return of the expropriated property, respondents would exhort on the pronouncement in Provincial Government
of Sorsogon vs. Vda. de Villaroya where the unpaid landowners were allowed the alternative remedy of recovery of the property there in
question. It might be borne in mind that the case involved the municipal government of Sorsogon, to which the power of eminent domain
is not inherent, but merely delegated and of limited application. The grant of the power of eminent domain to local governments under
Republic Act No. 7160 cannot be understood as being the pervasive and all-encompassing power vested in the legislative branch of
government. For local governments to be able to wield the power, it must, by enabling law, be delegated to it by the national legislature,
but even then, this delegated power of eminent domain is not, strictly speaking, a power of eminent, but only of inferior, domain or only
as broad or confined as the real authority would want it to be.

Thus, in Valdehueza vs. Republic where the private landowners had remained unpaid ten years after the termination of the expropriation
proceedings, this Court ruled -

“The points in dispute are whether such payment can still be made and, if so, in what amount. Said lots have been the subject of
expropriation proceedings. By final and executory judgment in said proceedings, they were condemned for public use, as part of an
airport, and ordered sold to the government. x x x It follows that both by virtue of the judgment, long final, in the expropriation suit, as
well as the annotations upon their title certificates, plaintiffs are not entitled to recover possession of their expropriated lots - which are
still devoted to the public use for which they were expropriated - but only to demand the fair market value of the same.

"Said relief may be granted under plaintiffs' prayer for: `such other remedies, which may be deemed just and equitable under the
premises'."

The Court proceeded to reiterate its pronouncement in Alfonso vs. Pasay City where the recovery of possession of property taken for
public use prayed for by the unpaid landowner was denied even while no requisite expropriation proceedings were first institu ted. The
landowner was merely given the relief of recovering compensation for his property computed at its market value at the time it was taken
and appropriated by the State.

The judgment rendered by the Bulacan RTC in 1979 on the expropriation proceedings provides not only for the payment of just
compensation to herein respondents but likewise adjudges the property condemned in favor of petitioner over which parties, as well as
their privies, are bound. Petitioner has occupied, utilized and, for all intents and purposes, exercised dominion over the property pursuant
to the judgment. The exercise of such rights vested to it as the condemnee indeed has amounted to at least a partial compliance or
satisfaction of the 1979 judgment, thereby preempting any claim of bar by prescription on grounds of non-execution. In arguing for the
return of their property on the basis of non-payment, respondents ignore the fact that the right of the expropriatory authority is far from
that of an unpaid seller in ordinary sales, to which the remedy of rescission might perhaps apply. An in rem proceeding, condemnation
acts upon the property. After condemnation, the paramount title is in the public under a new and independent title; thus, by giving notice
to all claimants to a disputed title, condemnation proceedings provide a judicial process for securing better title against all the world than
may be obtained by voluntary conveyance.

Respondents, in arguing laches against petitioner did not take into account that the same argument could likewise apply against them.
Respondents first instituted proceedings for payment against petitioner on 09 May 1984, or five years after the 1979 judgment had become
final. The unusually long delay in bringing the action to compel payment against herein petitioner would militate against them. Consistently
with the rule that one should take good care of his own concern, respondents should have commenced the proper action upon the finality
of the judgment which, indeed, resulted in a permanent deprivation of their ownership and possession of the property.

The constitutional limitation of “just compensation” is considered to be the sum equivalent to the market value of the property, broadly
described to be the price fixed by the seller in open market in the usual and ordinary course of legal action and competition or the fair
value of the property as between one who receives, and one who desires to sell, it fixed at the time of the actual taking by the government.
Thus, if property is taken for public use before compensation is deposited with the court having jurisdiction over the case, the final
compensation must include interests on its just value to be computed from the time the property is taken to the time when compensation
is actually paid or deposited with the court. In fine, between the taking of the property and the actual payment, legal interests accrue in
order to place the owner in a position as good as (but not better than) the position he was in before the taking occurred.

The Bulacan trial court, in its 1979 decision, was correct in imposing interests on the zonal value of the property to be computed from the
time petitioner instituted condemnation proceedings and “took” the property in September 1969. This allowance of interest on the amount
found to be the value of the property as of the time of the taking computed, being an effective forbearance, at 12% per annum should
help eliminate the issue of the constant fluctuation and inflation of the value of the currency over time. Article 1250 of the Civil Code,
providing that, in case of extraordinary inflation or deflation, the value of the currency at the time of the establishment of the obligation
shall be the basis for the payment when no agreement to the contrary is stipulated, has strict application only to contractual obligations.
In other words, a contractual agreement is needed for the effects of extraordinary inflation to be taken into account to alter the value of
the currency.
All given, the trial court of Bulacan in issuing its order, dated 01 March 2000, vacating its decision of 26 February 1979 has acted beyond
its lawful cognizance, the only authority left to it being to order its execution. Verily, private respondents, although not entitled to the return
of the expropriated property, deserve to be paid promptly on the yet unpaid award of just compensation already fixed by final judgment
of the Bulacan RTC on 26 February 1979 at P6.00 per square meter, with legal interest thereon at 12% per annum computed from the
date of "taking" of the property, i.e., 19 September 1969, until the due amount shall have been fully paid.
------------------------------------------XXX-----------------------------------------

Vous aimerez peut-être aussi